Evolution does not always optimize survival of an organism. Male moose evolved giant antlers as a way of fighting oth...

Zahra on July 20, 2018

Question 21

isn't the argument giving an example of the general claim that evolution does not always optimize survival? I ruled out C because it said it was giving a counterexample.

Replies
Create a free account to read and take part in forum discussions.

Already have an account? log in

Michelle on July 25, 2018

Hey @Zahra thanks for your question. This argument is actually challenging he general claim that "evolution always optimizes survival." In light of that, does it make sense why we would call the moose antler case a counter example of the general claim? Let me know if you have more questions about this!

Will on May 28, 2019

How is this not A? I can't figure out what is wrong with choice A because it is citing an example and its aim is to cast doubt on the competing argument that "evolution always optimizes survival".

Ravi on May 30, 2019

@wills,

Great question. Let's take a look at (A) and (C).

(A) says, "citing an example to cast doubt on a competing argument"

(C) says, "challenging a general claim by presenting a counterexample"

The key difference between (A) and (C) is whether or not the argument
is challenging/casting doubt on a competing argument (A) or general
claim (C).

The reason that (C) is correct and (A) is wrong is because the
argument is challenging a general claim. The general claim is
implicit—it's that some people believe that evolution always optimizes
for survival of an organism. The reason that this is a general
claim—and not an argument—is because an argument has at least two
parts: a premise and a conclusion. This claim does not have any
support (that is, a premise), so it's lacking the components necessary
to be considered an argument. This is why (A) is wrong.

Hope this helps. Let us know if you have any other questions!

on August 4, 2021

So A is wrong because it isn't an explicit (premise/conclusion) argument?

Ravi on February 6, 2022

@Alec, A is wrong because there aren't any competing arguments referenced, so it doesn't match up well.